Difference between revisions of "2006 Romanian NMO Problems/Grade 8/Problem 4"

m
(Solution)
 
Line 4: Line 4:
 
''selected by Mircea Lascu''
 
''selected by Mircea Lascu''
 
==Solution==
 
==Solution==
{{solution}}
+
 
 +
It is easy to see that the function <math>f(a,b,c)=\frac{a+b}{c+1}+\frac{b+c}{a+1}+\frac{c+a}{b+1}</math> is convex in each of the three variables (since each term is linear or of the form <math>\frac{p}{x+q}</math> for each variable <math>x</math>).  Thus, its value is maximized at the endpoints.  Checking the values of <math>f</math> for all possible values of <math>a,b,c</math> such that <math>a,b,c\in \{\frac{1}{2},1\}</math> yields a maximum of <math>3</math> as desired. 
 +
 
 +
As for the minimum, we have
 +
 
 +
<math>2\le \frac{a+b}{c+1}+\frac{b+c}{a+1}+\frac{c+a}{b+1}</math>
 +
 
 +
<math>\Leftrightarrow 5\le \frac{a+b+c+1}{c+1}+\frac{a+b+c+1}{a+1}+\frac{a+b+c+1}{b+1}</math>
 +
 
 +
Applying AM-HM to the right hand side yields
 +
 
 +
<math>9\left(\frac{a+b+c+3}{a+b+c+1}\right)^{-1}\le\frac{a+b+c+1}{c+1}+\frac{a+b+c+1}{a+1}+\frac{a+b+c+1}{b+1}</math>
 +
 
 +
<math>\Rightarrow 9\left(1-\frac{2}{a+b+c+3}\right)\le\frac{a+b+c+1}{c+1}+\frac{a+b+c+1}{a+1}+\frac{a+b+c+1}{b+1}</math>
 +
 
 +
Obviously, <math>\frac{2}{a+b+c+3}</math> is maximized when <math>a,b,c</math> are minimized.  That is, when <math>a=b=c=\frac{1}{2}</math>.  Thus, we have that
 +
 
 +
<math>5\le 9\left(1-\frac{2}{a+b+c+3}\right)</math>
 +
 
 +
as desired.
 +
 
 
==See also==
 
==See also==
 
*[[2006 Romanian NMO Problems/Grade 8/Problem 3 | Previous problem]]
 
*[[2006 Romanian NMO Problems/Grade 8/Problem 3 | Previous problem]]
 
*[[2006 Romanian NMO Problems]]
 
*[[2006 Romanian NMO Problems]]
 
[[Category:Olympiad Algebra Problems]]
 
[[Category:Olympiad Algebra Problems]]

Latest revision as of 19:49, 26 August 2008

Problem

Let $a,b,c \in \left[ \frac 12, 1 \right]$. Prove that

$2 \leq \frac{ a+b}{1+c} + \frac{ b+c}{1+a} + \frac{ c+a}{1+b} \leq 3$.

selected by Mircea Lascu

Solution

It is easy to see that the function $f(a,b,c)=\frac{a+b}{c+1}+\frac{b+c}{a+1}+\frac{c+a}{b+1}$ is convex in each of the three variables (since each term is linear or of the form $\frac{p}{x+q}$ for each variable $x$). Thus, its value is maximized at the endpoints. Checking the values of $f$ for all possible values of $a,b,c$ such that $a,b,c\in \{\frac{1}{2},1\}$ yields a maximum of $3$ as desired.

As for the minimum, we have

$2\le \frac{a+b}{c+1}+\frac{b+c}{a+1}+\frac{c+a}{b+1}$

$\Leftrightarrow 5\le \frac{a+b+c+1}{c+1}+\frac{a+b+c+1}{a+1}+\frac{a+b+c+1}{b+1}$

Applying AM-HM to the right hand side yields

$9\left(\frac{a+b+c+3}{a+b+c+1}\right)^{-1}\le\frac{a+b+c+1}{c+1}+\frac{a+b+c+1}{a+1}+\frac{a+b+c+1}{b+1}$

$\Rightarrow 9\left(1-\frac{2}{a+b+c+3}\right)\le\frac{a+b+c+1}{c+1}+\frac{a+b+c+1}{a+1}+\frac{a+b+c+1}{b+1}$

Obviously, $\frac{2}{a+b+c+3}$ is maximized when $a,b,c$ are minimized. That is, when $a=b=c=\frac{1}{2}$. Thus, we have that

$5\le 9\left(1-\frac{2}{a+b+c+3}\right)$

as desired.

See also